Saltar al contenido principal
LibreTexts Español

8: Método Gauss-Seidel

  • Page ID
    119391
    \( \newcommand{\vecs}[1]{\overset { \scriptstyle \rightharpoonup} {\mathbf{#1}} } \) \( \newcommand{\vecd}[1]{\overset{-\!-\!\rightharpoonup}{\vphantom{a}\smash {#1}}} \)\(\newcommand{\id}{\mathrm{id}}\) \( \newcommand{\Span}{\mathrm{span}}\) \( \newcommand{\kernel}{\mathrm{null}\,}\) \( \newcommand{\range}{\mathrm{range}\,}\) \( \newcommand{\RealPart}{\mathrm{Re}}\) \( \newcommand{\ImaginaryPart}{\mathrm{Im}}\) \( \newcommand{\Argument}{\mathrm{Arg}}\) \( \newcommand{\norm}[1]{\| #1 \|}\) \( \newcommand{\inner}[2]{\langle #1, #2 \rangle}\) \( \newcommand{\Span}{\mathrm{span}}\) \(\newcommand{\id}{\mathrm{id}}\) \( \newcommand{\Span}{\mathrm{span}}\) \( \newcommand{\kernel}{\mathrm{null}\,}\) \( \newcommand{\range}{\mathrm{range}\,}\) \( \newcommand{\RealPart}{\mathrm{Re}}\) \( \newcommand{\ImaginaryPart}{\mathrm{Im}}\) \( \newcommand{\Argument}{\mathrm{Arg}}\) \( \newcommand{\norm}[1]{\| #1 \|}\) \( \newcommand{\inner}[2]{\langle #1, #2 \rangle}\) \( \newcommand{\Span}{\mathrm{span}}\)\(\newcommand{\AA}{\unicode[.8,0]{x212B}}\)

    Objetivos de aprendizaje

    Después de leer este capítulo, deberías poder:

    1. resolver un conjunto de ecuaciones usando el método Gauss-Seidel,
    2. reconocer las ventajas y dificultades del método Gauss-Seidel, y
    3. determinar en qué condiciones converge siempre el método Gauss-Seidel.

    ¿Por qué necesitamos otro método para resolver un conjunto de ecuaciones lineales simultáneas?

    En ciertos casos, como cuando un sistema de ecuaciones es grande, los métodos iterativos de resolución de ecuaciones son más ventajosos. Los métodos de eliminación, como la eliminación gaussiana, son propensos a grandes errores de redondear para un gran conjunto de ecuaciones. Los métodos iterativos, como el método Gauss-Seidel, dan al usuario el control del error de redondear. Además, si la física del problema es bien conocida, las conjeturas iniciales necesarias en los métodos iterativos se pueden hacer más juiciosamente conduciendo a una convergencia más rápida.

    ¿Cuál es el algoritmo para el método Gauss-Seidel? Dado un conjunto general de\(n\) ecuaciones e\(n\) incógnitas, tenemos

    \[a_{11}x_{1} + a_{12}x_{2} + a_{13}x_{3} + ... + a_{1n}x_{n} = c_{1} \nonumber \]

    \[a_{21}x_{1} + a_{22}x_{2} + a_{23}x_{3} + ... + a_{2n}x_{n} = c_{2} \nonumber \]

    \[\vdots \ \ \ \ \ \ \ \ \ \ \ \ \ \ \ \ \ \ \ \vdots \nonumber \]

    \[a_{n1}x_{1} + a_{n2}x_{2} + a_{n3}x_{3} + ... + a_{nn}x_{n} = c_{n} \nonumber \]

    Si los elementos diagonales son distintos de cero, cada ecuación se reescribe para el desconocido correspondiente, es decir, la primera ecuación se reescribe con\(x_{1}\) en el lado izquierdo, la segunda ecuación se reescribe con\(x_{2}\) en el lado izquierdo y así sucesivamente de la siguiente manera

    \[x_{1} = \frac{c_{1} - a_{12}x_{2} - a_{13}x_{3}\ldots\ldots - a_{1n}x_{n}}{a_{11}} \nonumber \]

    \[x_{2} = \frac{c_{2} - a_{21}x_{1} - a_{23}x_{3}\ldots\ldots - a_{2n}x_{n}}{a_{22}} \nonumber \]

    \[\vdots \nonumber \]

    \[\vdots \nonumber \]

    \[x_{n - 1} = \frac{c_{n - 1} - a_{n - 1,1}x_{1} - a_{n - 1,2}x_{2}\ldots\ldots - a_{n - 1,n - 2}x_{n - 2} - a_{n - 1,n}x_{n}}{a_{n - 1,n - 1}} \nonumber \]

    \[x_{n} = \frac{c_{n} - a_{n1}x_{1} - a_{n2}x_{2} - \ldots\ldots - a_{n,n - 1}x_{n - 1}}{a_{nn}} \nonumber \]

    Estas ecuaciones se pueden reescribir en forma de suma como

    \[x_{1} = \frac{c_{1} - \sum_{\begin{matrix} j = 1 \\ j \neq 1 \\ \end{matrix}}^{n}{a_{1j}x_{j}}}{a_{11}} \nonumber \]

    \[x_{2} = \frac{c_{2} - \sum_{\begin{matrix} j = 1 \\ j \neq 2 \\ \end{matrix}}^{n}a_{2j}x_{j}}{a_{22}} \nonumber \]

    \[\vdots \nonumber \]

    \[x_{n - 1} = \frac{c_{n - 1} - \sum_{\begin{matrix} j = 1 \\ j \neq n - 1 \\ \end{matrix}}^{n}{a_{n - 1,j}x_{j}}}{a_{n - 1,n - 1}} \nonumber \]

    \[x_{n} = \frac{c_{n} - \sum_{\begin{matrix} j = 1 \\ j \neq n \\ \end{matrix}}^{n}{a_{nj}x_{j}}}{a_{nn}} \nonumber \]

    De ahí que para cualquier fila\(i\),

    \[x_{i} = \frac{c_{i} - \sum_{\begin{matrix} j = 1 \\ j \neq i \\ \end{matrix}}^{n}{a_{ij}x_{j}}}{a_{ii}},i = 1,2,\ldots,n. \nonumber \]

    Ahora para\(x_{i}\) encontrar's, uno asume una suposición inicial para los\(x_{i}\)'s y luego usa las ecuaciones reescritas para calcular las nuevas estimaciones. Recuerde, siempre se utilizan las estimaciones más recientes para calcular las próximas estimaciones,\(x_{i}\). Al final de cada iteración, se calcula el error aproximado relativo absoluto para cada una\(x_{i}\) como

    \[\left| \in_{a} \right|_{i} = \left| \frac{x_{i}^{\text{new}} - x_{i}^{\text{old}}}{x_{i}^{\text{new}}} \right| \times 100 \nonumber \]

    donde\(x_{i}^{\text{new}}\) es el valor obtenido recientemente de\(x_{i}\), y\(x_{i}^{\text{old}}\) es el valor anterior de\(x_{i}\).

    Cuando el error aproximado relativo absoluto para cada uno\(x_{i}\) es menor que la tolerancia preespecificada, las iteraciones se detienen.

    Ejemplo 1

    La velocidad ascendente de un cohete se da en tres momentos diferentes en la siguiente tabla

    Cuadro 1: Datos de velocidad vs. tiempo.

    Tiempo,\(t (s)\) Velocidad,\(v (m/s)\)
    \ (t (s)\) ">5 \ (v (m/s)\) ">106.8
    \ (t (s)\) ">8 \ (v (m/s)\) ">177.2
    \ (t (s)\) ">12 \ (v (m/s)\) ">279.2

    Los datos de velocidad son aproximados por un polinomio como

    \[v\left( t \right) = a_{1}t^{2} + a_{2}t + a_{3},5 \leq t \leq 12 \nonumber \]

    Encuentra los valores de\(a_{1},\ a_{2},and \ a_{3}\) usar el método Gauss-Seidel. Asumir una suposición inicial de la solución como

    \[\begin{bmatrix} a_{1} \\ a_{2} \\ a_{3} \\ \end{bmatrix} = \begin{bmatrix} 1 \\ 2 \\ 5 \\ \end{bmatrix} \nonumber \]

    y realizar dos iteraciones.

    Solución

    El polinomio está pasando por tres puntos de datos\(\left( t_{1},v_{1} \right),\left( t_{2},v_{2} \right),and\left( t_{3},v_{3} \right)\) donde de la tabla anterior

    \[t_{1} = 5,v_{1} = 106.8 \nonumber \]

    \[t_{2} = 8,v_{2} = 177.2 \nonumber \]

    \[t_{3} = 12,v_{3} = 279.2 \nonumber \]

    Requieren que\(v\left( t \right) = a_{1}t^{2} + a_{2}t + a_{3}\) pase por los tres puntos de datos da

    \[v\left( t_{1} \right) = v_{1} = a_{1}t_{1}^{2} + a_{2}t_{1} + a_{3} \nonumber \]

    \[v\left( t_{2} \right) = v_{2} = a_{1}t_{2}^{2} + a_{2}t_{2} + a_{3} \nonumber \]

    \[v\left( t_{3} \right) = v_{3} = a_{1}t_{3}^{2} + a_{2}t_{3} + a_{3} \nonumber \]

    Sustituir los datos\(\left( t_{1},v_{1} \right),\left( t_{2},v_{2} \right),and\left( t_{3},v_{3} \right)\) da

    \[a_{1}\left( 5^{2} \right) + a_{2}\left( 5 \right) + a_{3} = 106.8 \nonumber \]

    \[a_{1}\left( 8^{2} \right) + a_{2}\left( 8 \right) + a_{3} = 177.2 \nonumber \]

    \[a_{1}\left( 12^{2} \right) + a_{2}\left( 12 \right) + a_{3} = 279.2 \nonumber \]

    o

    \[25a_{1} + 5a_{2} + a_{3} = 106.8 \nonumber \]

    \[64a_{1} + 8a_{2} + a_{3} = 177.2 \nonumber \]

    \[144a_{1} + 12a_{2} + a_{3} = 279.2 \nonumber \]

    Los coeficientes\(a_{1},a_{2},anda_{3}\) para la expresión anterior vienen dados por

    \[\begin{bmatrix} 25 & 5 & 1 \\ 64 & 8 & 1 \\ 144 & 12 & 1 \\ \end{bmatrix}\begin{bmatrix} a_{1} \\ a_{2} \\ a_{3} \\ \end{bmatrix} = \begin{bmatrix} 106.8 \\ 177.2 \\ 279.2 \\ \end{bmatrix} \nonumber \]

    Reescribir las ecuaciones da

    \[a_{1} = \frac{106.8 - 5a_{2} - a_{3}}{25} \nonumber \]

    \[a_{2} = \frac{177.2 - 64a_{1} - a_{3}}{8} \nonumber \]

    \[a_{3} = \frac{279.2 - 144a_{1} - 12a_{2}}{1} \nonumber \]

    Iteración #1

    Dada la suposición inicial del vector de solución como

    \[\begin{bmatrix} a_{1} \\ a_{2} \\ a_{3} \\ \end{bmatrix} = \begin{bmatrix} 1 \\ 2 \\ 5 \\ \end{bmatrix} \nonumber \]

    obtenemos

    \[\begin{split} a_{1} &= \frac{106.8 - 5(2) - (5)}{25}\\ &= 3.6720 \end{split} \nonumber \]

    \[\begin{split} a_{2} &= \frac{177.2 - 64\left( 3.6720 \right) - \left( 5 \right)}{8}\\ &= - 7.8150 \end{split} \nonumber \]

    \[\begin{split} a_{3} &= \frac{279.2 - 144\left( 3.6720 \right) - 12\left( - 7.8510 \right)}{1}\\ &= - 155.36 \end{split} \nonumber \]

    El error aproximado relativo absoluto para cada\(x_{i}\) entonces es

    \[\begin{split} \left| \in_{a} \right|_{1} &= \left| \frac{3.6720 - 1}{3.6720} \right| \times 100\\ &= 72.76\% \end{split} \nonumber \]

    \[\begin{split} \left| \in_{a} \right|_{2} &= \left| \frac{- 7.8510 - 2}{- 7.8510} \right| \times 100\\ &= 125.47\% \end{split} \nonumber \]

    \[\begin{split} \left| \in_{a} \right|_{3} &= \left| \frac{- 155.36 - 5}{- 155.36} \right| \times 100\\ &= 103.22\% \end{split} \nonumber \]

    Al final de la primera iteración, la estimación del vector de solución es

    \[\begin{bmatrix} a_{1} \\ a_{2} \\ a_{3} \\ \end{bmatrix} = \begin{bmatrix} 3.6720 \\ - 7.8510 \\ - 155.36 \\ \end{bmatrix} \nonumber \]

    y el error aproximado relativo absoluto máximo es\(125.47%\).

    Iteración #2

    La estimación del vector de solución al final de la Iteración #1 es

    \[\begin{bmatrix} a_{1} \\ a_{2} \\ a_{3} \\ \end{bmatrix} = \begin{bmatrix} 3.6720 \\ - 7.8510 \\ - 155.36 \\ \end{bmatrix} \nonumber \]

    Ahora conseguimos

    \[\begin{split} a_{1} &= \frac{106.8 - 5\left( - 7.8510 \right) - ( - 155.36)}{25}\\ &= 12.056 \end{split} \nonumber \]

    \[\begin{split} a_{2} &= \frac{177.2 - 64\left( 12.056 \right) - ( - 155.36)}{8}\\ &= - 54.882 \end{split} \nonumber \]

    \[\begin{split} a_{3} &= \frac{279.2 - 144\left( 12.056 \right) - 12\left( - 54.882 \right)}{1}\\ &= - 798.34 \end{split} \nonumber \]

    El error aproximado relativo absoluto para cada\(x_{i}\) entonces es

    \[\begin{split} \left| \in_{a} \right|_{1} &= \left| \frac{12.056 - 3.6720}{12.056} \right| \times 100\\ &= 69.543\% \end{split} \nonumber \]

    \[\begin{split} \left| \in_{a} \right|_{2} &= \left| \frac{- 54.882 - \left( - 7.8510 \right)}{- 54.882} \right| \times 100\\ &= 85.695\% \end{split} \nonumber \]

    \[\begin{split} \left| \in_{a} \right|_{3} &= \left| \frac{- 798.34 - \left( - 155.36 \right)}{- 798.34} \right| \times 100\\ &= 80.540\% \end{split} \nonumber \]

    Al final de la segunda iteración, la estimación del vector de solución es

    \[\begin{bmatrix} a_{1} \\ a_{2} \\ a_{3} \\ \end{bmatrix} = \begin{bmatrix} 12.056 \\ - 54.882 \\ - 798.54 \\ \end{bmatrix} \nonumber \]

    y el error aproximado relativo absoluto máximo es\(85.695\%\).

    Al realizar más iteraciones se obtienen los siguientes valores para el vector de solución y los correspondientes errores aproximados relativos absolutos.

    Iteración \(a_{1}\) \(\left| \in_{a} \right|_{1} \%\) \(a_{2}\) \(\left| \in_{a} \right|_{2} \%\) \(a_{3}\) \(\left| \in_{a} \right|_{3} \%\)
    1 \ (a_ {1}\) ">3.672 \ (\ izquierda|\ in_ {a}\ derecha|_ {1}\%\) ">72.767 \ (a_ {2}\) ">—7.8510 \ (\ izquierda|\ in_ {a}\ derecha|_ {2}\%\) ">125.47 \ (a_ {3}\) ">—155.36 \ (\ izquierda|\ in_ {a}\ derecha|_ {3}\%\) ">103.22
    2 \ (a_ {1}\) ">12.056 \ (\ izquierda|\ in_ {a}\ derecha|_ {1}\%\) ">69.543 \ (a_ {2}\) ">—54.882 \ (\ izquierda|\ in_ {a}\ derecha|_ {2}\%\) ">85.695 \ (a_ {3}\) ">—798.34 \ (\ izquierda|\ in_ {a}\ derecha|_ {3}\%\) ">80.54
    3 \ (a_ {1}\) ">47.182 \ (\ izquierda|\ in_ {a}\ derecha|_ {1}\%\) ">74.447 \ (a_ {2}\) ">—255.51 \ (\ izquierda|\ in_ {a}\ derecha|_ {2}\%\) ">78.521 \ (a_ {3}\) ">—3448.9 \ (\ izquierda|\ in_ {a}\ derecha|_ {3}\%\) ">76.852
    4 \ (a_ {1}\) ">193.33 \ (\ izquierda|\ in_ {a}\ derecha|_ {1}\%\) ">75.595 \ (a_ {2}\) ">—1093.4 \ (\ izquierda|\ in_ {a}\ derecha|_ {2}\%\) ">76.632 \ (a_ {3}\) ">—14440 \ (\ izquierda|\ in_ {a}\ derecha|_ {3}\%\) ">76.116
    5 \ (a_ {1}\) ">800.53 \ (\ izquierda|\ in_ {a}\ derecha|_ {1}\%\) ">75.85 \ (a_ {2}\) ">—4577.2 \ (\ izquierda|\ in_ {a}\ derecha|_ {2}\%\) ">76.112 \ (a_ {3}\) ">—60072 \ (\ izquierda|\ in_ {a}\ derecha|_ {3}\%\) ">75.963
    6 \ (a_ {1}\) ">3322.6 \ (\ izquierda|\ in_ {a}\ derecha|_ {1}\%\) ">75.906 \ (a_ {2}\) ">—19049 \ (\ izquierda|\ in_ {a}\ derecha|_ {2}\%\) ">75.972 \ (a_ {3}\) ">—249580 \ (\ izquierda|\ in_ {a}\ derecha|_ {3}\%\) ">75.931

    Como se ve en la tabla anterior, las estimaciones de la solución no están convergiendo a la verdadera solución de

    \[a_{1} = 0.29048 \nonumber \]

    \[a_{2} = 19.690 \nonumber \]

    \[a_{3} = 1.0857 \nonumber \]

    El anterior sistema de ecuaciones no parece converger. ¿Por qué?

    Bueno, un escollo de la mayoría de los métodos iterativos es que pueden o no converger. Sin embargo, la solución a cierta clase de sistema de ecuaciones simultáneas siempre converge utilizando el método Gauss-Seidel. Esta clase de sistema de ecuaciones es donde la matriz de coeficientes\(\lbrack A\rbrack\) en\(\lbrack A\rbrack\lbrack X\rbrack = \lbrack C\rbrack\) es diagonalmente dominante, es decir

    \[\left| a_{ii} \right| \geq \sum_{\begin{matrix} j = 1 \\ j \neq i \\ \end{matrix}}^{n}\left| a_{ij} \right|\ \text{for all }i \nonumber \]

    \[\left| a_{ii} \right| > \sum_{\begin{matrix} j = 1 \\ j \neq i \\ \end{matrix}}^{n}\left| a_{ij} \right|\text{for at least one }i \nonumber \]

    Si un sistema de ecuaciones tiene una matriz de coeficientes que no es diagonalmente dominante, puede o no converger. Afortunadamente, muchos sistemas físicos que dan como resultado ecuaciones lineales simultáneas tienen una matriz de coeficientes diagonalmente dominante, que luego asegura la convergencia para métodos iterativos como el método Gauss-Seidel para resolver ecuaciones lineales simultáneas.

    Ejemplo 2

    Encuentre la solución al siguiente sistema de ecuaciones utilizando el método Gauss-Seidel.

    \[12x_{1} + 3x_{2} - 5x_{3} = 1 \nonumber \]

    \[x_{1} + 5x_{2} + 3x_{3} = 28 \nonumber \]

    \[3x_{1} + 7x_{2} + 13x_{3} = 76 \nonumber \]

    Uso

    \[\begin{bmatrix} x_{1} \\ x_{2} \\ x_{3} \\ \end{bmatrix} = \begin{bmatrix} 1 \\ 0 \\ 1 \\ \end{bmatrix} \nonumber \]

    como la suposición inicial y realizar dos iteraciones.

    Solución1

    La matriz de coeficientes

    \[\left\lbrack A \right\rbrack = \begin{bmatrix} 12 & 3 & - 5 \\ 1 & 5 & 3 \\ 3 & 7 & 13 \\ \end{bmatrix} \nonumber \]

    es diagonalmente dominante como

    \[\left| a_{11} \right| = \left| 12 \right| = 12 \geq \left| a_{12} \right| + \left| a_{13} \right| = \left| 3 \right| + \left| - 5 \right| = 8 \nonumber \]

    \[\left| a_{22} \right| = \left| 5 \right| = 5 \geq \left| a_{21} \right| + \left| a_{23} \right| = \left| 1 \right| + \left| 3 \right| = 4 \nonumber \]

    \[\left| a_{33} \right| = \left| 13 \right| = 13 \geq \left| a_{31} \right| + \left| a_{32} \right| = \left| 3 \right| + \left| 7 \right| = 10 \nonumber \]

    y la desigualdad es estrictamente mayor que para al menos una fila. Por lo tanto, la solución debería converger utilizando el método Gauss-Seidel.

    Reescribiendo las ecuaciones, obtenemos

    \[x_{1} = \frac{1 - 3x_{2} + 5x_{3}}{12} \nonumber \]

    \[x_{2} = \frac{28 - x_{1} - 3x_{3}}{5} \nonumber \]

    \[x_{3} = \frac{76 - 3x_{1} - 7x_{2}}{13} \nonumber \]

    Asumiendo una suposición inicial de

    \[\begin{bmatrix} x_{1} \\ x_{2} \\ x_{3} \\ \end{bmatrix} = \begin{bmatrix} 1 \\ 0 \\ 1 \\ \end{bmatrix} \nonumber \]

    Iteración #1

    \[\begin{split} x_{1} &= \frac{1 - 3\left( 0 \right) + 5\left( 1 \right)}{12}\\ & = 0.50000 \end{split} \nonumber \]

    \[\begin{split} x_{2} &= \frac{28 - \left( 0.50000 \right) - 3\left( 1 \right)}{5}\\ &= 4.9000 \end{split} \nonumber \]

    \[\begin{split} x_{3} &= \frac{76 - 3\left( 0.50000 \right) - 7\left( 4.9000 \right)}{13}\\ &= 3.0923 \end{split} \nonumber \]

    El error aproximado relativo absoluto al final de la primera iteración es

    \[\begin{split} \left| \in_{a} \right|_{1} &= \left| \frac{0.50000 - 1}{0.50000} \right| \times 100\\ &= 100.00\% \end{split} \nonumber \]

    \[\begin{split} \left| \in_{a} \right|_{2} &= \left| \frac{4.9000 - 0}{4.9000} \right| \times 100\\ &= 100.00\% \end{split} \nonumber \]

    \[\begin{split} \left| \in_{a} \right|_{3} &= \left| \frac{3.0923 - 1}{3.0923} \right| \times 100\\ &= 67.662\% \end{split} \nonumber \]

    El error aproximado relativo absoluto máximo es\(100.00\%\)

    Iteración #2

    \[\begin{split} x_{1} &= \frac{1 - 3\left( 4.9000 \right) + 5\left( 3.0923 \right)}{12}\\ &= 0.14679 \end{split} \nonumber \]

    \[\begin{split} x_{2} &= \frac{28 - \left( 0.14679 \right) - 3\left( 3.0923 \right)}{5}\\ &= 3.7153 \end{split} \nonumber \]

    \[\begin{split} x_{3} &= \frac{76 - 3\left( 0.14679 \right) - 7\left( 3.7153 \right)}{13}\\ &= 3.8118 \end{split} \nonumber \]

    Al final de la segunda iteración, el error aproximado relativo absoluto es

    \[\begin{split} \left| \in_{a} \right|_{1} &= \left| \frac{0.14679 - 0.50000}{0.14679} \right| \times 100\\ &= 240.61\% \end{split} \nonumber \]

    \[\begin{split} \left| \in_{a} \right|_{2} &= \left| \frac{3.7153 - 4.9000}{3.7153} \right| \times 100\\ &= 31.889\% \end{split} \nonumber \]

    \[\begin{split} \left| \in_{a} \right|_{3} &= \left| \frac{3.8118 - 3.0923}{3.8118} \right| \times 100\\ &= 18.874\% \end{split} \nonumber \]

    El error aproximado relativo absoluto máximo es\(240.61\%\). Esto es mayor que el valor de\(100.00\%\) lo que obtuvimos en la primera iteración. ¿La solución es divergente? No, a medida que realiza más iteraciones, la solución converge de la siguiente manera.

    Iteración \(a_{1}\) \(\left| \in_{a} \right|_{1} \%\) \(a_{2}\) \(\left| \in_{a} \right|_{2} \%\) \(a_{3}\) \(\left| \in_{a} \right|_{3} \%\)
    1 \ (a_ {1}\) ">0.5 \ (\ izquierda|\ in_ {a}\ derecha|_ {1}\%\) ">100 \ (a_ {2}\) ">4.9 \ (\ izquierda|\ in_ {a}\ derecha|_ {2}\%\) ">100 \ (a_ {3}\) ">3.0923 \ (\ izquierda|\ in_ {a}\ derecha|_ {3}\%\) ">67.662
    2 \ (a_ {1}\) ">0.14679 \ (\ izquierda|\ in_ {a}\ derecha|_ {1}\%\) ">240.61 \ (a_ {2}\) ">3.7153 \ (\ izquierda|\ in_ {a}\ derecha|_ {2}\%\) ">31.889 \ (a_ {3}\) ">3.8118 \ (\ izquierda|\ in_ {a}\ derecha|_ {3}\%\) ">18.874
    3 \ (a_ {1}\) ">0.74275 \ (\ izquierda|\ in_ {a}\ derecha|_ {1}\%\) ">80.236 \ (a_ {2}\) ">3.1644 \ (\ izquierda|\ in_ {a}\ derecha|_ {2}\%\) ">17.408 \ (a_ {3}\) ">3.9708 \ (\ izquierda|\ in_ {a}\ derecha|_ {3}\%\) ">4.0064
    4 \ (a_ {1}\) ">0.94675 \ (\ izquierda|\ in_ {a}\ derecha|_ {1}\%\) ">21.546 \ (a_ {2}\) ">3.0281 \ (\ izquierda|\ in_ {a}\ derecha|_ {2}\%\) ">4.4996 \ (a_ {3}\) ">3.9971 \ (\ izquierda|\ in_ {a}\ derecha|_ {3}\%\) ">0.65772
    5 \ (a_ {1}\) ">0.99177 \ (\ izquierda|\ in_ {a}\ derecha|_ {1}\%\) ">4.5391 \ (a_ {2}\) ">3.0034 \ (\ izquierda|\ in_ {a}\ derecha|_ {2}\%\) ">0.82499 \ (a_ {3}\) ">4.0001 \ (\ izquierda|\ in_ {a}\ derecha|_ {3}\%\) ">0.074383
    6 \ (a_ {1}\) ">0.99919 \ (\ izquierda|\ in_ {a}\ derecha|_ {1}\%\) ">0.74307 \ (a_ {2}\) ">3.0001 \ (\ izquierda|\ in_ {a}\ derecha|_ {2}\%\) ">0.10856 \ (a_ {3}\) ">4.0001 \ (\ izquierda|\ in_ {a}\ derecha|_ {3}\%\) ">0.00101

    Esto está cerca del vector de solución exacto de

    \[\begin{bmatrix} x_{1} \\ x_{2} \\ x_{3} \\ \end{bmatrix} = \begin{bmatrix} 1 \\ 3 \\ 4 \\ \end{bmatrix} \nonumber \]

    Ejemplo 3

    Dado el sistema de ecuaciones

    \[3x_{1} + 7x_{2} + 13x_{3} = 76 \nonumber \]

    \[x_{1} + 5x_{2} + 3x_{3} = 28 \nonumber \]

    \[12x_{1} + 3x_{2} - 5x_{3} = 1 \nonumber \]

    encuentre la solución usando el método Gauss-Seidel. Uso

    \[\begin{bmatrix} x_{1} \\ x_{2} \\ x_{3} \\ \end{bmatrix} = \begin{bmatrix} 1 \\ 0 \\ 1 \\ \end{bmatrix} \nonumber \]

    como suposición inicial.

    Solución1

    Reescribiendo las ecuaciones, obtenemos

    \[x_{1} = \frac{76 - 7x_{2} - 13x_{3}}{3} \nonumber \]

    \[x_{2} = \frac{28 - x_{1} - 3x_{3}}{5} \nonumber \]

    \[x_{3} = \frac{1 - 12x_{1} - 3x_{2}}{- 5} \nonumber \]

    Asumiendo una suposición inicial de

    \[\begin{bmatrix} x_{1} \\ x_{2} \\ x_{3} \\ \end{bmatrix} = \begin{bmatrix} 1 \\ 0 \\ 1 \\ \end{bmatrix} \nonumber \]

    los siguientes seis valores iterativos se dan en la siguiente tabla.

    Iteración \(a_{1}\) \(\left| \in_{a} \right|_{1} \%\) \(a_{2}\) \(\left| \in_{a} \right|_{2} \%\) \(a_{3}\) \(\left| \in_{a} \right|_{3} \%\)
    1 \ (a_ {1}\) ">21 \ (\ izquierda|\ in_ {a}\ derecha|_ {1}\%\) ">95.238 \ (a_ {2}\) ">0.8 \ (\ izquierda|\ in_ {a}\ derecha|_ {2}\%\) ">100 \ (a_ {3}\) ">50.68 \ (\ izquierda|\ in_ {a}\ derecha|_ {3}\%\) ">98.027
    2 \ (a_ {1}\) ">—196.15 \ (\ izquierda|\ in_ {a}\ derecha|_ {1}\%\) ">110.71 \ (a_ {2}\) ">14.421 \ (\ izquierda|\ in_ {a}\ derecha|_ {2}\%\) ">94.453 \ (a_ {3}\) ">—462.30 \ (\ izquierda|\ in_ {a}\ derecha|_ {3}\%\) ">110.96
    3 \ (a_ {1}\) ">1995 \ (\ izquierda|\ in_ {a}\ derecha|_ {1}\%\) ">109.83 \ (a_ {2}\) ">—116.02 \ (\ izquierda|\ in_ {a}\ derecha|_ {2}\%\) ">112.43 \ (a_ {3}\) ">4718.1 \ (\ izquierda|\ in_ {a}\ derecha|_ {3}\%\) ">109.8
    4 \ (a_ {1}\) ">—20149 \ (\ izquierda|\ in_ {a}\ derecha|_ {1}\%\) ">109.9 \ (a_ {2}\) ">1204.6 \ (\ izquierda|\ in_ {a}\ derecha|_ {2}\%\) ">109.63 \ (a_ {3}\) ">—47636 \ (\ izquierda|\ in_ {a}\ derecha|_ {3}\%\) ">109.9
    5 \ (a_ {1}\) ">2.0364\(\times 10^5\) \ (\ izquierda|\ in_ {a}\ derecha|_ {1}\%\) ">109.89 \ (a_ {2}\) ">—12140 \ (\ izquierda|\ in_ {a}\ derecha|_ {2}\%\) ">109.92 \ (a_ {3}\) ">4.8144\(\times 10^5\) \ (\ izquierda|\ in_ {a}\ derecha|_ {3}\%\) ">109.89
    6 \ (a_ {1}\) ">—2.0579\(\times 10^6\) \ (\ izquierda|\ in_ {a}\ derecha|_ {1}\%\) ">109.89 \ (a_ {2}\) ">1.2272\(\times 10^5\) \ (\ izquierda|\ in_ {a}\ derecha|_ {2}\%\) ">109.89 \ (a_ {3}\) ">—4.8653\(\times 10^6\) \ (\ izquierda|\ in_ {a}\ derecha|_ {3}\%\) ">109.89

    Se puede ver que esta solución no es convergente y la matriz de coeficientes no es diagonalmente dominante. La matriz de coeficientes

    \[\left\lbrack A \right\rbrack = \begin{bmatrix} 3 & 7 & 13 \\ 1 & 5 & 3 \\ 12 & 3 & - 5 \\ \end{bmatrix} \nonumber \]

    no es diagonalmente dominante como

    \[\left| a_{11} \right| = \left| 3 \right| = 3 \leq \left| a_{12} \right| + \left| a_{13} \right| = \left| 7 \right| + \left| 13 \right| = 20 \nonumber \]

    De ahí que el método Gauss-Seidel pueda o no converger.

    Sin embargo, es el mismo conjunto de ecuaciones que el ejemplo anterior y que convergió. La única diferencia es que intercambiamos la primera y la tercera ecuación entre sí y eso hizo que la matriz de coeficientes no fuera diagonalmente dominante.

    Por lo tanto, es posible que un sistema de ecuaciones pueda hacerse diagonalmente dominante si se intercambian las ecuaciones entre sí. Sin embargo, no es posible para todos los casos. Por ejemplo, el siguiente conjunto de ecuaciones

    \[x_{1} + x_{2} + x_{3} = 3 \nonumber \]

    \[2x_{1} + 3x_{2} + 4x_{3} = 9 \nonumber \]

    \[x_{1} + 7x_{2} + x_{3} = 9 \nonumber \]

    no se puede reescribir para hacer que la matriz de coeficientes sea diagonalmente dominante.

    Cuestionario del método Gauss-Seidel

    Quiz 1

    Una matriz cuadrada\(\left\lbrack A \right\rbrack_{n \times n}\) es diagonalmente dominante si

    (A)\(\displaystyle \left| a_{ii} \right| \geq \sum_{\begin{matrix} j = 1 \\ i \neq j \\ \end{matrix}}^{n}\left| a_{ij} \right|\),\(i = 1,2,...,n\)

    (B)\(\displaystyle \left| a_{ii} \right| \geq \sum_{\begin{matrix} j = 1 \\ i \neq j \\ \end{matrix}}^{n}\left| a_{ij} \right|,\)\(i = 1,2,...,n\) y\(\left| a_{ii} \right| > \sum_{\begin{matrix} j = 1 \\ i \neq j \\ \end{matrix}}^{n}\left| a_{ij} \right|,\) para cualquier\(i = 1,2,...,n\)

    (C)\(\displaystyle \left| a_{ii} \right| \geq \sum_{j = 1}^{n}\left| a_{ij} \right|,\)\(i = 1,2,...,n\) y\(\left| a_{ii} \right| > \sum_{j = 1}^{n}\left| a_{ij} \right|,\) para cualquier\(i = 1,2,...,n\)

    (D)\(\displaystyle \left| a_{ii} \right| \geq \sum_{j = 1}^{n}\left| a_{ij} \right|,\)\(i = 1,2,...,n\)

    Quiz 2

    Usando\(\lbrack x_{1},x_{2},x_{3}\rbrack = \lbrack 1,3,5\rbrack\) como suposición inicial, los valores de\(\lbrack x_{1},x_{2},x_{3}\rbrack\) después de tres iteraciones en el método Gauss-Seidel para

    \[\begin{bmatrix} 12 & 7 & 3 \\ 1 & 5 & 1 \\ 2 & 7 & - 11 \\ \end{bmatrix}\begin{bmatrix} x_{1} \\ x_{2} \\ x_{3} \\ \end{bmatrix} = \begin{bmatrix} 2 \\ - 5 \\ 6 \\ \end{bmatrix} \nonumber \]

    son

    (A)\([-2.8333 -1.4333 -1.9727]\)

    (B)\([1.4959 -0.90464 -0.84914]\)

    (C)\([0.90666 -1.0115 -1.0243]\)

    (D)\([1.2148 -0.72060 -0.82451]\)

    Quiz 3

    Para asegurar que el siguiente sistema de ecuaciones,

    \[\begin{matrix} 2x_{1} + & 7x_{2} - & 11x_{3} = & 6 \\ x_{1} + & 2x_{2} + & x_{3} = & - 5 \\ 7x_{1} + & 5x_{2} + & 2x_{3} = & 17 \\ \end{matrix} \nonumber \]

    converge usando el método Gauss-Seidel, se pueden reescribir las ecuaciones anteriores de la siguiente manera:

    (A)\(\begin{bmatrix} 2 & 7 & - 11 \\ 1 & 2 & 1 \\ 7 & 5 & 2 \\ \end{bmatrix}\ \begin{bmatrix} x_{1} \\ x_{2} \\ x_{3} \\ \end{bmatrix} = \begin{bmatrix} 6 \\ -5 \\ 17 \\ \end{bmatrix}\)

    (B)\(\begin{bmatrix} 7 & 5 & 2 \\ 1 & 2 & 1 \\ 2 & 7 & - 11 \\ \end{bmatrix}\ \begin{bmatrix} x_{1} \\ x_{2} \\ x_{3} \\ \end{bmatrix} = \begin{bmatrix} 17 \\ -5 \\ 6 \\ \end{bmatrix}\)

    (C)\(\begin{bmatrix} 7 & 5 & 2 \\ 1 & 2 & 1 \\ 2 & 7 & - 11 \\ \end{bmatrix}\ \begin{bmatrix} x_{1} \\ x_{2} \\ x_{3} \\ \end{bmatrix} = \begin{bmatrix} 6 \\ -5 \\ 17 \\ \end{bmatrix}\)

    (D) Las ecuaciones no pueden ser reescritas en una forma que asegure la convergencia.

    Quiz 4

    Para\(\begin{bmatrix} 12 & 7 & 3 \\ 1 & 5 & 1 \\ 2 & 7 & - 11 \\ \end{bmatrix}\ \begin{bmatrix} x_{1} \\ x_{2} \\ x_{3} \\ \end{bmatrix} = \begin{bmatrix} 22 \\ 7 \\ - 2 \\ \end{bmatrix}\) y usando\(\begin{bmatrix} x_{1} & x_{2} & x_{3} \\ \end{bmatrix} = \begin{bmatrix} 1 & 2 & 1 \\ \end{bmatrix}\) como suposición inicial, los valores de\(\begin{bmatrix} x_{1} & x_{2} & x_{3} \\ \end{bmatrix}\) se encuentran al final de cada iteración como

    Iteración # \(x_{1}\) \(x_{2}\) \(x_{3}\)
    1 \ (x_ {1}\) ">0.41667 \ (x_ {2}\) ">1.1167 \ (x_ {3}\) ">0.96818
    2 \ (x_ {1}\) ">0.93990 \ (x_ {2}\) ">1.0184 \ (x_ {3}\) ">1.0008
    3 \ (x_ {1}\) ">0.98908 \ (x_ {2}\) ">1.0020 \ (x_ {3}\) ">0.99931
    4 \ (x_ {1}\) ">0.99899 \ (x_ {2}\) ">1.0003 \ (x_ {3}\) ">1.0000

    ¿En qué primer número de iteración confiarías al menos en 1 dígito significativo en tu solución?

    (A)\(1\)

    (B)\(2\)

    (C)\(3\)

    (D)\(4\)

    Quiz 5

    El algoritmo para resolver el método Gauss-Seidel\(\left\lbrack A \right\rbrack\left\lbrack X \right\rbrack = \left\lbrack C \right\rbrack\) se da de la siguiente manera cuando se usan\(n\max\) iteraciones. El valor inicial de\(\left\lbrack X \right\rbrack\) se almacena en\(\left\lbrack X \right\rbrack\).

    (A) Sub

    Seidel\((n,a,x,{rhs},nmax)\)

    \(k = 1\)Para\({nmax}\)

    \(i = 1\)Para\(n\)

    \(j = 1\)Para\(n\)

    Si (\(i < > j\)) Entonces

    Suma = Suma +\(a(i,j)*x(j)\)

    endif

    Siguiente\(j\)

    \(x(i) = ( {rhs}(i) - \text{Sum})/a(i,i)\)

    Siguiente\(i\)

    Siguiente\(j\)

    Fin Sub

    (B) Sub

    Seidel\((n,a,x,{rhs},nmax)\)

    \(k = 1\)Para\({nmax}\)

    \(i = 1\)Para\(n\)

    Suma = 0

    \(j = 1\)Para\(n\)

    Si (\(i < > j\)) Entonces

    Suma = Suma +\(a(i,j)*x(j)\)

    endif

    Siguiente\(j\)

    \(x(i) = ( {rhs}(i) - \text{Sum})/a(i,i)\)

    Siguiente\(i\)

    Siguiente\(k\)

    Fin Sub

    (C) Sub

    Seidel\((n,a,x,{rhs},nmax)\)

    \(k = 1\)Para\({nmax}\)

    \(i = 1\)Para\(n\)

    Suma = 0

    \(j = 1\)Para\(n\)

    Suma = Suma +\(a(i,j)*x(j)\)

    Siguiente\(j\)

    \(x(i) = ( {rhs}(i) - \text{Sum})/a(i,i)\)

    Siguiente\(i\)

    Siguiente\(k\)

    Fin Sub

    (D) Sub

    Seidel\((n,a,x,{rhs},nmax)\)

    \(k = 1\)Para {nmax} $

    \(i = 1\)Para\(n\)

    Suma = 0

    \(j = 1\)Para\(n\)

    Si (\(i < > j\)) Entonces

    Suma = Suma +\(a(i,j)*x(j)\)

    endif

    Siguiente\(j\)

    \(x(i) = ( {rhs}(i) - \text{Sum})/a(i,i)\)

    Siguiente\(i\)

    Siguiente\(k\)

    Fin Sub

    Quiz 6

    Los termistores miden la temperatura, tienen una salida no lineal y se valoran para un rango limitado. Entonces, cuando se fabrica un termistor, el fabricante suministra una curva de resistencia vs. temperatura. Una representación precisa de la curva generalmente viene dada por

    \[\frac{1}{T} = a_{0} + a_{1}ln(R) + a_{2}\left\{ \ln\left( R \right) \right\}^{2} + a_{3}\left\{ \ln\left( R \right) \right\}^{3} \nonumber \]

    donde\(T\) es la temperatura en Kelvin,\(R\) es la resistencia en ohmios, y\(a_{0},a_{1},a_{2},a_{3}\) son constantes de la curva de calibración. Dado lo siguiente para un termistor

    \(R\) \(T\)
    \ (R\) ">ohmios \ (T\) ">\({^\circ}C\)
    \ (R\) ">

    1101.0

    911.3

    636.0

    451.1

    \ (T\) ">

    25.113

    30.131

    40.120

    50.128

    el valor de la temperatura en\({^\circ}C\) para una resistencia medida de\(900\) ohmios más cerca es

    (A)\(30.002\)

    (B)\(30.473\)

    (C)\(31.272\)

    (D)\(31.445\)

    Ejercicio del Método Gauss-Seidel

    Ejercicio 1

    En un sistema de ecuación\([A] [X] = [C]\), si\([A]\) es diagonalmente dominante, entonces el método Gaussseidal-seidel

    1. siempre converge
    2. puede o no converger
    3. siempre diverge
    Responder

    A

    Ejercicio 2

    En un sistema de ecuaciones\([A] [X] = [C]\), si no\([A]\) es diagonalmente dominante, entonces el método Gauss-Seidel

    1. Siempre converge
    2. Puede o no converger
    3. Siempre diverge.
    Responder

    B

    Ejercicio 3

    En un sistema de ecuaciones\([A] [X] = [C]\), si no\([A]\) es diagonalmente dominante, el sistema de ecuaciones siempre se puede reescribir para hacerlo diagonalmente dominante.

    1. Cierto
    2. Falso
    Responder

    B

    Ejercicio 4

    Resuelve el siguiente sistema de ecuaciones usando el método Gauss-Seidel

    \[\begin{matrix} 12x_{1} + 7x_{2} + 3x_{3} = 2 \\ x_{1} + 5x_{2} + x_{3} = - 5 \\ 2x_{1} + 7x_{2} - 11x_{3} = 6 \\ \end{matrix} \nonumber \]

    Realiza 3 iteraciones, calcula el error aproximado relativo absoluto máximo al final de cada iteración y elige\(\begin{bmatrix} x_{1} & x_{2} & x_{3} \\ \end{bmatrix} = \begin{bmatrix} 1 & 3 & 5 \\ \end{bmatrix}\) como tu suposición inicial.

    Responder

    \(\begin{bmatrix} x_{1} & x_{2} & x_{3} \\ \end{bmatrix} = \begin{bmatrix} 0.90666 & -1.0115 & -1.0243 \\ \end{bmatrix}\)\({\begin{bmatrix} \left| \in_{a} \right|_{1} & \left| \in_{a} \right|_{2} & \left| \in_{a} \right|_{3} \\ \end{bmatrix} = \begin{bmatrix} 65.001\% & 10.564\% & 17.099\% \\ \end{bmatrix} }\)\({\begin{bmatrix} \left| \in_{a} \right|_{1} & \left| \in_{a} \right|_{2} & \left| \in_{a} \right|_{3} \\ \end{bmatrix} = \begin{bmatrix} 65.001\% & 10.564\% & 17.099\% \\ \end{bmatrix} }\)

    Ejercicio 5

    Resuelve el siguiente sistema de ecuaciones usando el método Gauss-Seidel

    \(12x_{1} + 7x_{2} + 3x_{3} = 2\)
    \(x_{1} + 5x_{2} + x_{3} = - 5\)
    \(2x_{1} + 7x_{2} - 11x_{3} = 6\)
    Realice 3 iteraciones, calcule el error aproximado relativo absoluto máximo al final de cada iteración y elija\(\begin{bmatrix} x_{1} & x_{2} & x_{3} \\ \end{bmatrix} = \begin{bmatrix} 1 & 3 & 5 \\ \end{bmatrix}\) como su suposición inicial.

    Responder

    \(\begin{bmatrix} x_{1} & x_{2} & x_{3} \\ \end{bmatrix} = \begin{bmatrix} 0.90666 & - 1.0115 & - 1.0243 \\ \end{bmatrix}\)

    Ejercicio 6

    Resuelva el siguiente sistema de ecuaciones usando el método Gauss-Seidel
    \(x_{1} + 5x_{2} + x_{3} = 5\)
    \(12x_{1} + 7x_{2} + 3x_{3} = 2\)
    \(2x_{1} + 7x_{2} - 11x_{3} = 6\)
    Realice 3 iteraciones, calcule el error aproximado relativo absoluto máximo al final de cada iteración, y elige\(\begin{bmatrix} x_{1} & x_{2} & x_{3} \\ \end{bmatrix} = \begin{bmatrix} 1 & 3 & 5 \\ \end{bmatrix}\) como tu suposición inicial.

    Responder

    \(\begin{bmatrix} x_{1} & x_{2} & x_{3} \\ \end{bmatrix} = \begin{bmatrix} -1163.7 & 1947.6 & 1027.2 \\ \end{bmatrix}\)

    \(\begin{bmatrix} \left| \in_{a} \right|_{1} & \left| \in_{a} \right|_{2} & \left| \in_{a} \right|_{3} \\ \end{bmatrix} = \begin{bmatrix} 89.156\% & 89.139\% & 89.183\% \\ \end{bmatrix}\)


    This page titled 8: Método Gauss-Seidel is shared under a CC BY-NC-SA 4.0 license and was authored, remixed, and/or curated by Autar Kaw via source content that was edited to the style and standards of the LibreTexts platform; a detailed edit history is available upon request.